Solving a linear equation with a cross product

Click For Summary
The discussion centers on solving the equation αv + (a × v) = b for the vector v, where α is a scalar and a, b are fixed vectors. The user has derived two key identities by manipulating the equation but is struggling to progress further. A suggestion is made to analyze the components of v, specifically separating it into components along and perpendicular to the vector a × b. This approach aims to simplify the problem by utilizing the geometric properties of the vectors involved. The conversation highlights the challenges of using dot and cross products in vector equations and seeks guidance on advancing the solution.
SP90
Messages
19
Reaction score
0

Homework Statement



Suppose v is a vector satisfying:

\alpha v + ( a \times v ) = b

For \alpha a scalar and a, b fixed vectors. Use dot and cross product operations to solve the above for v.

Homework Equations



The unique solutions should be:

v=\frac{\alpha^{2}b- \alpha (b \times a) + (b \cdot a)a}{\alpha(\alpha^{2}+|a|^{2}})

I'm having trouble getting there.

The Attempt at a Solution



I get that ( a \times v ) = b - \alpha v and then dotting both sides with a and v gives two identities:

(b - \alpha v) \cdot a = 0
(b - \alpha v) \cdot v = 0

Which rearrange to give

(b \cdot a) = \alpha (v \cdot a)
(b \cdot a) = \alpha (v \cdot v) = \alpha |v|^{2}

I don't know where to go from here. There's no inverse cross product and I've tried several different combinations of cross and dot products which lead to dead ends, or are just the same equations as those two I've just derived.

Any help would be much appreciated.
 
Physics news on Phys.org
Hi SP90! :smile:

You have three vectors, a b and axb, with the third perpendicular to the other two.

So start by finding the component of v along axb, and the component of v perpendicular to axb. :wink:
 
Question: A clock's minute hand has length 4 and its hour hand has length 3. What is the distance between the tips at the moment when it is increasing most rapidly?(Putnam Exam Question) Answer: Making assumption that both the hands moves at constant angular velocities, the answer is ## \sqrt{7} .## But don't you think this assumption is somewhat doubtful and wrong?

Similar threads

Replies
4
Views
2K
  • · Replies 5 ·
Replies
5
Views
2K
  • · Replies 13 ·
Replies
13
Views
4K
Replies
8
Views
2K
  • · Replies 4 ·
Replies
4
Views
2K
Replies
2
Views
1K
Replies
15
Views
2K
Replies
3
Views
2K
  • · Replies 6 ·
Replies
6
Views
2K
  • · Replies 6 ·
Replies
6
Views
4K